Question and Answers Forum

All Questions      Topic List

Mechanics Questions

Previous in All Question      Next in All Question      

Previous in Mechanics      Next in Mechanics      

Question Number 108619 by ajfour last updated on 18/Aug/20

Commented by ajfour last updated on 18/Aug/20

Find tension in rod connecting  the blocks.

$${Find}\:{tension}\:{in}\:{rod}\:{connecting} \\ $$$${the}\:{blocks}. \\ $$

Commented by Dwaipayan Shikari last updated on 18/Aug/20

For upper block  T+mgsinθ−f_1 =ma       (f=μN=μmgcosθ)  (θ=tan^(−1) (8/(15)))  Lower block  mgsinθ−T−f_2 =ma  T+mgsinθ−f_1 =mgsinθ−T−f_2   2T=f_1 −f_2   T=((μ_1 mgcosθ−μ_2 mgcosθ)/2)=(1/2)mgcosθ(0.2)=(g/(10)).170.cosθ=g.17.((15)/(17))N                                                                                                                           =15gN

$${For}\:{upper}\:{block} \\ $$$${T}+{mgsin}\theta−{f}_{\mathrm{1}} ={ma}\:\:\:\:\:\:\:\left({f}=\mu{N}=\mu{mgcos}\theta\right)\:\:\left(\theta={tan}^{−\mathrm{1}} \frac{\mathrm{8}}{\mathrm{15}}\right) \\ $$$${Lower}\:{block} \\ $$$${mgsin}\theta−{T}−{f}_{\mathrm{2}} ={ma} \\ $$$${T}+{mgsin}\theta−{f}_{\mathrm{1}} ={mgsin}\theta−{T}−{f}_{\mathrm{2}} \\ $$$$\mathrm{2}{T}={f}_{\mathrm{1}} −{f}_{\mathrm{2}} \\ $$$${T}=\frac{\mu_{\mathrm{1}} {mgcos}\theta−\mu_{\mathrm{2}} {mgcos}\theta}{\mathrm{2}}=\frac{\mathrm{1}}{\mathrm{2}}{mgcos}\theta\left(\mathrm{0}.\mathrm{2}\right)=\frac{{g}}{\mathrm{10}}.\mathrm{170}.{cos}\theta={g}.\mathrm{17}.\frac{\mathrm{15}}{\mathrm{17}}{N} \\ $$$$\:\:\:\:\:\:\:\:\:\:\:\:\:\:\:\:\:\:\:\:\:\:\:\:\:\:\:\:\:\:\:\:\:\:\:\:\:\:\:\:\:\:\:\:\:\:\:\:\:\:\:\:\:\:\:\:\:\:\:\:\:\:\:\:\:\:\:\:\:\:\:\:\:\:\:\:\:\:\:\:\:\:\:\:\:\:\:\:\:\:\:\:\:\:\:\:\:\:\:\:\:\:\:\:\:\:\:\:\:\:\:\:\:\:\:\:\:\:\:\:\:=\mathrm{15}{gN} \\ $$$$ \\ $$$$ \\ $$

Commented by ajfour last updated on 18/Aug/20

Nice way, thanks Sir.

$${Nice}\:{way},\:{thanks}\:{Sir}. \\ $$

Answered by mr W last updated on 18/Aug/20

a=acceleration  a=((2mg sin θ−mg(μ_1 +μ_2 )cos θ)/(2m))  =[sin θ−(((μ_1 +μ_2 )cos θ)/2)]g >0    T=mg sin θ−μ_2 mg cos θ−ma  =mg[sin θ−μ_2 cos θ−((2sin θ−(μ_1 +μ_2 )cos θ)/2)]  =((mg(μ_1 −μ_2 )cos θ)/2)

$${a}={acceleration} \\ $$$${a}=\frac{\mathrm{2}{mg}\:\mathrm{sin}\:\theta−{mg}\left(\mu_{\mathrm{1}} +\mu_{\mathrm{2}} \right)\mathrm{cos}\:\theta}{\mathrm{2}{m}} \\ $$$$=\left[\mathrm{sin}\:\theta−\frac{\left(\mu_{\mathrm{1}} +\mu_{\mathrm{2}} \right)\mathrm{cos}\:\theta}{\mathrm{2}}\right]{g}\:>\mathrm{0} \\ $$$$ \\ $$$${T}={mg}\:\mathrm{sin}\:\theta−\mu_{\mathrm{2}} {mg}\:\mathrm{cos}\:\theta−{ma} \\ $$$$={mg}\left[\mathrm{sin}\:\theta−\mu_{\mathrm{2}} \mathrm{cos}\:\theta−\frac{\mathrm{2sin}\:\theta−\left(\mu_{\mathrm{1}} +\mu_{\mathrm{2}} \right)\mathrm{cos}\:\theta}{\mathrm{2}}\right] \\ $$$$=\frac{{mg}\left(\mu_{\mathrm{1}} −\mu_{\mathrm{2}} \right)\mathrm{cos}\:\theta}{\mathrm{2}} \\ $$

Commented by ajfour last updated on 18/Aug/20

Thanks Sir, right answer; 150N.

$${Thanks}\:{Sir},\:{right}\:{answer};\:\mathrm{150}{N}. \\ $$

Terms of Service

Privacy Policy

Contact: info@tinkutara.com